Auto-correlation of a stochastic equation

  • Thread starter Thread starter Tyler_D
  • Start date Start date
  • Tags Tags
    Stochastic
Click For Summary
The discussion focuses on the auto-correlation of a stochastic equation involving Wiener integrals and Brownian motion. The author attempts to derive an expression for the correlation function but becomes uncertain about how to handle the exponential terms and whether to use Taylor expansion. There is confusion regarding the notation and limits of integration, particularly about the relationship between W(t) and W(t+τ). The conversation also touches on the independence of integrals and the expectation values involved. The author acknowledges a typo in the limits of integration and seeks clarification on the properties of the Wiener integral.
Tyler_D
Messages
6
Reaction score
0
Homework Statement
Calculate the auto-correlation of ##x(t)=e^{-\beta W(t)}W(t)##
Relevant Equations
##\langle x(t)x(t+\tau)\rangle##, ##W(t)=\int_0^t dW(s)##
Here is my attempt:

$$
\begin{align}
\langle x(t)x(t+\tau) \rangle &= \langle (e^{-\beta W(t)}W(t))(e^{-\beta W(t+\tau)}W(t+\tau))\rangle \\
&= \langle (e^{-\beta W(t)}e^{-\beta W(t+\tau)})(W(t)W(t+\tau))\rangle \\
&= \langle (e^{-\beta W(t)}e^{-\beta W(t+\tau)})(\int_0^t dW(t)\int_0^{t+\tau}dW(t))\rangle \\
&= \langle (e^{-\beta W(t)}e^{-\beta W(t+\tau)})\int_0^t dW(t)(\int_0^{t}dW(t)+\int_t^{\tau}dW(t))\rangle
\end{align}
$$

From here on I am quite lost on what to do with the exponentials. Do I Taylor-expand them, or what is the best thing to do?
 
Physics news on Phys.org
Is W(t) some special function? The Lambert W function maybe? That would make nice integrals.
 
It is the Wiener-integral/Brownian motion (my bad, should've written it in the OP):

##
W(t) = \int_0^t dW(s)
##
 
Last edited:
I don't understand something, you wrote in Eq. (4) ##\int_0^{t+\tau}dW(s) = \int_0^t dW(s)+\int_t^{\tau}dW(s)##, but obviously this means that: ##\int_0^{t+\tau}dW(s)=\int_0^\tau dW(s)##.
Does this mean that Weiner integral is periodic with period ##t##, since then ##W(t+\tau)=W(\tau)##?
 
Last edited:
  • Like
Likes Tyler_D
That's also a typo, thanks for spotting it. The upper limit in the last integral should naturally be ##\tau+t##, not just ##\tau##.
 
Question: A clock's minute hand has length 4 and its hour hand has length 3. What is the distance between the tips at the moment when it is increasing most rapidly?(Putnam Exam Question) Answer: Making assumption that both the hands moves at constant angular velocities, the answer is ## \sqrt{7} .## But don't you think this assumption is somewhat doubtful and wrong?

Similar threads

Replies
2
Views
2K
  • · Replies 1 ·
Replies
1
Views
1K
  • · Replies 4 ·
Replies
4
Views
2K
  • · Replies 1 ·
Replies
1
Views
1K
Replies
1
Views
2K
  • · Replies 1 ·
Replies
1
Views
1K
Replies
4
Views
2K
  • · Replies 2 ·
Replies
2
Views
1K
  • · Replies 3 ·
Replies
3
Views
2K
  • · Replies 1 ·
Replies
1
Views
2K